Is it true that $lim_{x to a}frac{f(x)-f(a)}{x-a}=pminfty implies lim_{x to a}f'(x) = pminfty$?












6












$begingroup$


Is it true that $lim_{x to a}frac{f(x)-f(a)}{x-a}=pminfty implies lim_{x to a}f'(x) = pminfty$?



Here, $f$ is a function defined on some open interval $I$, and $ain I$. Assume $f$ is continuous at $a$ and differentiable around $a$.



I can't for the life of me see how to provedisprove this implication, but my gut feeling is that it's false. Any guidance is greatly appreciated.










share|cite|improve this question











$endgroup$












  • $begingroup$
    isn't that definition of limit? the (ε, δ) should provide that the function is strictly increasing and must go unbounded to $ infty $
    $endgroup$
    – user29418
    Jan 7 at 1:28










  • $begingroup$
    Derivative of a differential function need not be continuous
    $endgroup$
    – Sorfosh
    Jan 7 at 6:00
















6












$begingroup$


Is it true that $lim_{x to a}frac{f(x)-f(a)}{x-a}=pminfty implies lim_{x to a}f'(x) = pminfty$?



Here, $f$ is a function defined on some open interval $I$, and $ain I$. Assume $f$ is continuous at $a$ and differentiable around $a$.



I can't for the life of me see how to provedisprove this implication, but my gut feeling is that it's false. Any guidance is greatly appreciated.










share|cite|improve this question











$endgroup$












  • $begingroup$
    isn't that definition of limit? the (ε, δ) should provide that the function is strictly increasing and must go unbounded to $ infty $
    $endgroup$
    – user29418
    Jan 7 at 1:28










  • $begingroup$
    Derivative of a differential function need not be continuous
    $endgroup$
    – Sorfosh
    Jan 7 at 6:00














6












6








6


2



$begingroup$


Is it true that $lim_{x to a}frac{f(x)-f(a)}{x-a}=pminfty implies lim_{x to a}f'(x) = pminfty$?



Here, $f$ is a function defined on some open interval $I$, and $ain I$. Assume $f$ is continuous at $a$ and differentiable around $a$.



I can't for the life of me see how to provedisprove this implication, but my gut feeling is that it's false. Any guidance is greatly appreciated.










share|cite|improve this question











$endgroup$




Is it true that $lim_{x to a}frac{f(x)-f(a)}{x-a}=pminfty implies lim_{x to a}f'(x) = pminfty$?



Here, $f$ is a function defined on some open interval $I$, and $ain I$. Assume $f$ is continuous at $a$ and differentiable around $a$.



I can't for the life of me see how to provedisprove this implication, but my gut feeling is that it's false. Any guidance is greatly appreciated.







calculus






share|cite|improve this question















share|cite|improve this question













share|cite|improve this question




share|cite|improve this question








edited Jan 7 at 1:14









amWhy

192k28225439




192k28225439










asked Jan 7 at 0:32









Euler's FriendEuler's Friend

415




415












  • $begingroup$
    isn't that definition of limit? the (ε, δ) should provide that the function is strictly increasing and must go unbounded to $ infty $
    $endgroup$
    – user29418
    Jan 7 at 1:28










  • $begingroup$
    Derivative of a differential function need not be continuous
    $endgroup$
    – Sorfosh
    Jan 7 at 6:00


















  • $begingroup$
    isn't that definition of limit? the (ε, δ) should provide that the function is strictly increasing and must go unbounded to $ infty $
    $endgroup$
    – user29418
    Jan 7 at 1:28










  • $begingroup$
    Derivative of a differential function need not be continuous
    $endgroup$
    – Sorfosh
    Jan 7 at 6:00
















$begingroup$
isn't that definition of limit? the (ε, δ) should provide that the function is strictly increasing and must go unbounded to $ infty $
$endgroup$
– user29418
Jan 7 at 1:28




$begingroup$
isn't that definition of limit? the (ε, δ) should provide that the function is strictly increasing and must go unbounded to $ infty $
$endgroup$
– user29418
Jan 7 at 1:28












$begingroup$
Derivative of a differential function need not be continuous
$endgroup$
– Sorfosh
Jan 7 at 6:00




$begingroup$
Derivative of a differential function need not be continuous
$endgroup$
– Sorfosh
Jan 7 at 6:00










1 Answer
1






active

oldest

votes


















8












$begingroup$

The claim is false. Consider
$$
g(x)=sqrt{x}sinfrac{1}{x}+x^{1/4},qquad x>0,
$$

and define $f$ on $mathbb R$ by
$$
f(x)=begin{cases}g(x) & text{for }x>0, \ 0 & text{for }x=0, \ -g(-x) & text{for }x<0.end{cases}
$$

Then $f$ is continuous everywhere, differentiable in $mathbb Rbackslash{0}$ and satisfies $f(-x)=-f(x)$.
Moreover, for $x>0$,
begin{align*}
frac{f(x)}{x}=frac{1}{sqrt{x}}sinfrac{1}{x}+frac{1}{x^{3/4}}geq-frac{1}{x^{1/2}}+frac{1}{x^{3/4}}longrightarrow+infty,quadtext{as }xto0+.
end{align*}

Due to the symmetry, the same is true for $x<0$ and $xto0-$. Thus, $f(x)/xto+infty$ as $xto0$.



Now, for $x>0$,
$$
f'(x)=frac{x^{3/4}+2 x sin left(frac{1}{x}right)-4 cos left(frac{1}{x}right)}{4 x^{3/2}}.
$$

However, the limit $lim_{xto0+}f'(x)$ does not even exist. For $x_n:=1/(npi)$ we have
begin{align*}
f'(x_n)=frac{pi ^{3/4}}{4 left(frac{1}{n}right)^{3/4}}-frac{pi ^{3/2} (-1)^n}{left(frac{1}{n}right)^{3/2}},
end{align*}

and so $f'(x_{2n})to-infty$, while $f'(x_{2n+1})to+infty$.






share|cite|improve this answer









$endgroup$













    Your Answer





    StackExchange.ifUsing("editor", function () {
    return StackExchange.using("mathjaxEditing", function () {
    StackExchange.MarkdownEditor.creationCallbacks.add(function (editor, postfix) {
    StackExchange.mathjaxEditing.prepareWmdForMathJax(editor, postfix, [["$", "$"], ["\\(","\\)"]]);
    });
    });
    }, "mathjax-editing");

    StackExchange.ready(function() {
    var channelOptions = {
    tags: "".split(" "),
    id: "69"
    };
    initTagRenderer("".split(" "), "".split(" "), channelOptions);

    StackExchange.using("externalEditor", function() {
    // Have to fire editor after snippets, if snippets enabled
    if (StackExchange.settings.snippets.snippetsEnabled) {
    StackExchange.using("snippets", function() {
    createEditor();
    });
    }
    else {
    createEditor();
    }
    });

    function createEditor() {
    StackExchange.prepareEditor({
    heartbeatType: 'answer',
    autoActivateHeartbeat: false,
    convertImagesToLinks: true,
    noModals: true,
    showLowRepImageUploadWarning: true,
    reputationToPostImages: 10,
    bindNavPrevention: true,
    postfix: "",
    imageUploader: {
    brandingHtml: "Powered by u003ca class="icon-imgur-white" href="https://imgur.com/"u003eu003c/au003e",
    contentPolicyHtml: "User contributions licensed under u003ca href="https://creativecommons.org/licenses/by-sa/3.0/"u003ecc by-sa 3.0 with attribution requiredu003c/au003e u003ca href="https://stackoverflow.com/legal/content-policy"u003e(content policy)u003c/au003e",
    allowUrls: true
    },
    noCode: true, onDemand: true,
    discardSelector: ".discard-answer"
    ,immediatelyShowMarkdownHelp:true
    });


    }
    });














    draft saved

    draft discarded


















    StackExchange.ready(
    function () {
    StackExchange.openid.initPostLogin('.new-post-login', 'https%3a%2f%2fmath.stackexchange.com%2fquestions%2f3064553%2fis-it-true-that-lim-x-to-a-fracfx-fax-a-pm-infty-implies-lim%23new-answer', 'question_page');
    }
    );

    Post as a guest















    Required, but never shown

























    1 Answer
    1






    active

    oldest

    votes








    1 Answer
    1






    active

    oldest

    votes









    active

    oldest

    votes






    active

    oldest

    votes









    8












    $begingroup$

    The claim is false. Consider
    $$
    g(x)=sqrt{x}sinfrac{1}{x}+x^{1/4},qquad x>0,
    $$

    and define $f$ on $mathbb R$ by
    $$
    f(x)=begin{cases}g(x) & text{for }x>0, \ 0 & text{for }x=0, \ -g(-x) & text{for }x<0.end{cases}
    $$

    Then $f$ is continuous everywhere, differentiable in $mathbb Rbackslash{0}$ and satisfies $f(-x)=-f(x)$.
    Moreover, for $x>0$,
    begin{align*}
    frac{f(x)}{x}=frac{1}{sqrt{x}}sinfrac{1}{x}+frac{1}{x^{3/4}}geq-frac{1}{x^{1/2}}+frac{1}{x^{3/4}}longrightarrow+infty,quadtext{as }xto0+.
    end{align*}

    Due to the symmetry, the same is true for $x<0$ and $xto0-$. Thus, $f(x)/xto+infty$ as $xto0$.



    Now, for $x>0$,
    $$
    f'(x)=frac{x^{3/4}+2 x sin left(frac{1}{x}right)-4 cos left(frac{1}{x}right)}{4 x^{3/2}}.
    $$

    However, the limit $lim_{xto0+}f'(x)$ does not even exist. For $x_n:=1/(npi)$ we have
    begin{align*}
    f'(x_n)=frac{pi ^{3/4}}{4 left(frac{1}{n}right)^{3/4}}-frac{pi ^{3/2} (-1)^n}{left(frac{1}{n}right)^{3/2}},
    end{align*}

    and so $f'(x_{2n})to-infty$, while $f'(x_{2n+1})to+infty$.






    share|cite|improve this answer









    $endgroup$


















      8












      $begingroup$

      The claim is false. Consider
      $$
      g(x)=sqrt{x}sinfrac{1}{x}+x^{1/4},qquad x>0,
      $$

      and define $f$ on $mathbb R$ by
      $$
      f(x)=begin{cases}g(x) & text{for }x>0, \ 0 & text{for }x=0, \ -g(-x) & text{for }x<0.end{cases}
      $$

      Then $f$ is continuous everywhere, differentiable in $mathbb Rbackslash{0}$ and satisfies $f(-x)=-f(x)$.
      Moreover, for $x>0$,
      begin{align*}
      frac{f(x)}{x}=frac{1}{sqrt{x}}sinfrac{1}{x}+frac{1}{x^{3/4}}geq-frac{1}{x^{1/2}}+frac{1}{x^{3/4}}longrightarrow+infty,quadtext{as }xto0+.
      end{align*}

      Due to the symmetry, the same is true for $x<0$ and $xto0-$. Thus, $f(x)/xto+infty$ as $xto0$.



      Now, for $x>0$,
      $$
      f'(x)=frac{x^{3/4}+2 x sin left(frac{1}{x}right)-4 cos left(frac{1}{x}right)}{4 x^{3/2}}.
      $$

      However, the limit $lim_{xto0+}f'(x)$ does not even exist. For $x_n:=1/(npi)$ we have
      begin{align*}
      f'(x_n)=frac{pi ^{3/4}}{4 left(frac{1}{n}right)^{3/4}}-frac{pi ^{3/2} (-1)^n}{left(frac{1}{n}right)^{3/2}},
      end{align*}

      and so $f'(x_{2n})to-infty$, while $f'(x_{2n+1})to+infty$.






      share|cite|improve this answer









      $endgroup$
















        8












        8








        8





        $begingroup$

        The claim is false. Consider
        $$
        g(x)=sqrt{x}sinfrac{1}{x}+x^{1/4},qquad x>0,
        $$

        and define $f$ on $mathbb R$ by
        $$
        f(x)=begin{cases}g(x) & text{for }x>0, \ 0 & text{for }x=0, \ -g(-x) & text{for }x<0.end{cases}
        $$

        Then $f$ is continuous everywhere, differentiable in $mathbb Rbackslash{0}$ and satisfies $f(-x)=-f(x)$.
        Moreover, for $x>0$,
        begin{align*}
        frac{f(x)}{x}=frac{1}{sqrt{x}}sinfrac{1}{x}+frac{1}{x^{3/4}}geq-frac{1}{x^{1/2}}+frac{1}{x^{3/4}}longrightarrow+infty,quadtext{as }xto0+.
        end{align*}

        Due to the symmetry, the same is true for $x<0$ and $xto0-$. Thus, $f(x)/xto+infty$ as $xto0$.



        Now, for $x>0$,
        $$
        f'(x)=frac{x^{3/4}+2 x sin left(frac{1}{x}right)-4 cos left(frac{1}{x}right)}{4 x^{3/2}}.
        $$

        However, the limit $lim_{xto0+}f'(x)$ does not even exist. For $x_n:=1/(npi)$ we have
        begin{align*}
        f'(x_n)=frac{pi ^{3/4}}{4 left(frac{1}{n}right)^{3/4}}-frac{pi ^{3/2} (-1)^n}{left(frac{1}{n}right)^{3/2}},
        end{align*}

        and so $f'(x_{2n})to-infty$, while $f'(x_{2n+1})to+infty$.






        share|cite|improve this answer









        $endgroup$



        The claim is false. Consider
        $$
        g(x)=sqrt{x}sinfrac{1}{x}+x^{1/4},qquad x>0,
        $$

        and define $f$ on $mathbb R$ by
        $$
        f(x)=begin{cases}g(x) & text{for }x>0, \ 0 & text{for }x=0, \ -g(-x) & text{for }x<0.end{cases}
        $$

        Then $f$ is continuous everywhere, differentiable in $mathbb Rbackslash{0}$ and satisfies $f(-x)=-f(x)$.
        Moreover, for $x>0$,
        begin{align*}
        frac{f(x)}{x}=frac{1}{sqrt{x}}sinfrac{1}{x}+frac{1}{x^{3/4}}geq-frac{1}{x^{1/2}}+frac{1}{x^{3/4}}longrightarrow+infty,quadtext{as }xto0+.
        end{align*}

        Due to the symmetry, the same is true for $x<0$ and $xto0-$. Thus, $f(x)/xto+infty$ as $xto0$.



        Now, for $x>0$,
        $$
        f'(x)=frac{x^{3/4}+2 x sin left(frac{1}{x}right)-4 cos left(frac{1}{x}right)}{4 x^{3/2}}.
        $$

        However, the limit $lim_{xto0+}f'(x)$ does not even exist. For $x_n:=1/(npi)$ we have
        begin{align*}
        f'(x_n)=frac{pi ^{3/4}}{4 left(frac{1}{n}right)^{3/4}}-frac{pi ^{3/2} (-1)^n}{left(frac{1}{n}right)^{3/2}},
        end{align*}

        and so $f'(x_{2n})to-infty$, while $f'(x_{2n+1})to+infty$.







        share|cite|improve this answer












        share|cite|improve this answer



        share|cite|improve this answer










        answered Jan 7 at 2:27









        sranthropsranthrop

        7,0241925




        7,0241925






























            draft saved

            draft discarded




















































            Thanks for contributing an answer to Mathematics Stack Exchange!


            • Please be sure to answer the question. Provide details and share your research!

            But avoid



            • Asking for help, clarification, or responding to other answers.

            • Making statements based on opinion; back them up with references or personal experience.


            Use MathJax to format equations. MathJax reference.


            To learn more, see our tips on writing great answers.




            draft saved


            draft discarded














            StackExchange.ready(
            function () {
            StackExchange.openid.initPostLogin('.new-post-login', 'https%3a%2f%2fmath.stackexchange.com%2fquestions%2f3064553%2fis-it-true-that-lim-x-to-a-fracfx-fax-a-pm-infty-implies-lim%23new-answer', 'question_page');
            }
            );

            Post as a guest















            Required, but never shown





















































            Required, but never shown














            Required, but never shown












            Required, but never shown







            Required, but never shown

































            Required, but never shown














            Required, but never shown












            Required, but never shown







            Required, but never shown







            Popular posts from this blog

            Mario Kart Wii

            What does “Dominus providebit” mean?

            Antonio Litta Visconti Arese